Show that the $l^p$ norms with $1leq p,qleq infty$ are equivalent












1












$begingroup$


I want to show that for any $1leq p,qleq infty$, that there exists $a,b$ such that $aleft|xright|_p leq left|xright|_q leq bleft|xright|_p$ for all $x$ in $mathbb{R}^n$.

Is there a way to do this without Hölder's Inequalities?










share|cite|improve this question











$endgroup$












  • $begingroup$
    I am able to do this in the case of the 2-norm and infinity norm with a =1 and b =sqrt(n), is the proof similar to this specific case?
    $endgroup$
    – Sam.S
    Jan 16 at 22:31










  • $begingroup$
    Yes: it is quite elementary when $p=infty$, and the general case follows.
    $endgroup$
    – Mindlack
    Jan 16 at 22:32






  • 1




    $begingroup$
    What is $X$ here?
    $endgroup$
    – Lorenzo Quarisa
    Jan 16 at 22:34










  • $begingroup$
    Its the set of R^n
    $endgroup$
    – Sam.S
    Jan 16 at 22:36










  • $begingroup$
    Well, we do have to be careful. When e.g., $q =infty$ and $p=1$ the constants $a$ and $b$ may depend on $n$. I would imagine that the OP wants $a$ and $b$ to be of the form $a=a(p,q)$ and $b=b(p,q)$ where both $a,a^{-1},b,b^{-1}$ are functions of $p$ and $q$ only--no matter how large $n$ may be
    $endgroup$
    – Mike
    Jan 16 at 22:47


















1












$begingroup$


I want to show that for any $1leq p,qleq infty$, that there exists $a,b$ such that $aleft|xright|_p leq left|xright|_q leq bleft|xright|_p$ for all $x$ in $mathbb{R}^n$.

Is there a way to do this without Hölder's Inequalities?










share|cite|improve this question











$endgroup$












  • $begingroup$
    I am able to do this in the case of the 2-norm and infinity norm with a =1 and b =sqrt(n), is the proof similar to this specific case?
    $endgroup$
    – Sam.S
    Jan 16 at 22:31










  • $begingroup$
    Yes: it is quite elementary when $p=infty$, and the general case follows.
    $endgroup$
    – Mindlack
    Jan 16 at 22:32






  • 1




    $begingroup$
    What is $X$ here?
    $endgroup$
    – Lorenzo Quarisa
    Jan 16 at 22:34










  • $begingroup$
    Its the set of R^n
    $endgroup$
    – Sam.S
    Jan 16 at 22:36










  • $begingroup$
    Well, we do have to be careful. When e.g., $q =infty$ and $p=1$ the constants $a$ and $b$ may depend on $n$. I would imagine that the OP wants $a$ and $b$ to be of the form $a=a(p,q)$ and $b=b(p,q)$ where both $a,a^{-1},b,b^{-1}$ are functions of $p$ and $q$ only--no matter how large $n$ may be
    $endgroup$
    – Mike
    Jan 16 at 22:47
















1












1








1


0



$begingroup$


I want to show that for any $1leq p,qleq infty$, that there exists $a,b$ such that $aleft|xright|_p leq left|xright|_q leq bleft|xright|_p$ for all $x$ in $mathbb{R}^n$.

Is there a way to do this without Hölder's Inequalities?










share|cite|improve this question











$endgroup$




I want to show that for any $1leq p,qleq infty$, that there exists $a,b$ such that $aleft|xright|_p leq left|xright|_q leq bleft|xright|_p$ for all $x$ in $mathbb{R}^n$.

Is there a way to do this without Hölder's Inequalities?







inequality norm






share|cite|improve this question















share|cite|improve this question













share|cite|improve this question




share|cite|improve this question








edited Jan 16 at 22:42







Sam.S

















asked Jan 16 at 22:30









Sam.SSam.S

679




679












  • $begingroup$
    I am able to do this in the case of the 2-norm and infinity norm with a =1 and b =sqrt(n), is the proof similar to this specific case?
    $endgroup$
    – Sam.S
    Jan 16 at 22:31










  • $begingroup$
    Yes: it is quite elementary when $p=infty$, and the general case follows.
    $endgroup$
    – Mindlack
    Jan 16 at 22:32






  • 1




    $begingroup$
    What is $X$ here?
    $endgroup$
    – Lorenzo Quarisa
    Jan 16 at 22:34










  • $begingroup$
    Its the set of R^n
    $endgroup$
    – Sam.S
    Jan 16 at 22:36










  • $begingroup$
    Well, we do have to be careful. When e.g., $q =infty$ and $p=1$ the constants $a$ and $b$ may depend on $n$. I would imagine that the OP wants $a$ and $b$ to be of the form $a=a(p,q)$ and $b=b(p,q)$ where both $a,a^{-1},b,b^{-1}$ are functions of $p$ and $q$ only--no matter how large $n$ may be
    $endgroup$
    – Mike
    Jan 16 at 22:47




















  • $begingroup$
    I am able to do this in the case of the 2-norm and infinity norm with a =1 and b =sqrt(n), is the proof similar to this specific case?
    $endgroup$
    – Sam.S
    Jan 16 at 22:31










  • $begingroup$
    Yes: it is quite elementary when $p=infty$, and the general case follows.
    $endgroup$
    – Mindlack
    Jan 16 at 22:32






  • 1




    $begingroup$
    What is $X$ here?
    $endgroup$
    – Lorenzo Quarisa
    Jan 16 at 22:34










  • $begingroup$
    Its the set of R^n
    $endgroup$
    – Sam.S
    Jan 16 at 22:36










  • $begingroup$
    Well, we do have to be careful. When e.g., $q =infty$ and $p=1$ the constants $a$ and $b$ may depend on $n$. I would imagine that the OP wants $a$ and $b$ to be of the form $a=a(p,q)$ and $b=b(p,q)$ where both $a,a^{-1},b,b^{-1}$ are functions of $p$ and $q$ only--no matter how large $n$ may be
    $endgroup$
    – Mike
    Jan 16 at 22:47


















$begingroup$
I am able to do this in the case of the 2-norm and infinity norm with a =1 and b =sqrt(n), is the proof similar to this specific case?
$endgroup$
– Sam.S
Jan 16 at 22:31




$begingroup$
I am able to do this in the case of the 2-norm and infinity norm with a =1 and b =sqrt(n), is the proof similar to this specific case?
$endgroup$
– Sam.S
Jan 16 at 22:31












$begingroup$
Yes: it is quite elementary when $p=infty$, and the general case follows.
$endgroup$
– Mindlack
Jan 16 at 22:32




$begingroup$
Yes: it is quite elementary when $p=infty$, and the general case follows.
$endgroup$
– Mindlack
Jan 16 at 22:32




1




1




$begingroup$
What is $X$ here?
$endgroup$
– Lorenzo Quarisa
Jan 16 at 22:34




$begingroup$
What is $X$ here?
$endgroup$
– Lorenzo Quarisa
Jan 16 at 22:34












$begingroup$
Its the set of R^n
$endgroup$
– Sam.S
Jan 16 at 22:36




$begingroup$
Its the set of R^n
$endgroup$
– Sam.S
Jan 16 at 22:36












$begingroup$
Well, we do have to be careful. When e.g., $q =infty$ and $p=1$ the constants $a$ and $b$ may depend on $n$. I would imagine that the OP wants $a$ and $b$ to be of the form $a=a(p,q)$ and $b=b(p,q)$ where both $a,a^{-1},b,b^{-1}$ are functions of $p$ and $q$ only--no matter how large $n$ may be
$endgroup$
– Mike
Jan 16 at 22:47






$begingroup$
Well, we do have to be careful. When e.g., $q =infty$ and $p=1$ the constants $a$ and $b$ may depend on $n$. I would imagine that the OP wants $a$ and $b$ to be of the form $a=a(p,q)$ and $b=b(p,q)$ where both $a,a^{-1},b,b^{-1}$ are functions of $p$ and $q$ only--no matter how large $n$ may be
$endgroup$
– Mike
Jan 16 at 22:47












1 Answer
1






active

oldest

votes


















1












$begingroup$

$|x|_{infty} leq 1$ implies $|x|_{p} leq n^{1/p}$ for any $p in [1,infty)$. Use this and a scaling argument to show that $|x|_{p} leq n^{1/p} |x|_{infty}$. [You have to consider $y=frac x {|x|_{infty}}$ when $x neq 0$]. On the other hand, $ |x|_{infty} leq |x|_{p} $ because $|x_i| leq |x|_{p} $ for each $i$. Hence all of the $p$-norms with $p<infty$ are equivalent to $infty$ norm. This implies that any two of the $p$-norms are equivalent. [ $|x|' leq c|x|''$ and $|x|'' leq d|x|'''$ implies $|x|' leq cd|x|'''$ etc].






share|cite|improve this answer











$endgroup$













    Your Answer





    StackExchange.ifUsing("editor", function () {
    return StackExchange.using("mathjaxEditing", function () {
    StackExchange.MarkdownEditor.creationCallbacks.add(function (editor, postfix) {
    StackExchange.mathjaxEditing.prepareWmdForMathJax(editor, postfix, [["$", "$"], ["\\(","\\)"]]);
    });
    });
    }, "mathjax-editing");

    StackExchange.ready(function() {
    var channelOptions = {
    tags: "".split(" "),
    id: "69"
    };
    initTagRenderer("".split(" "), "".split(" "), channelOptions);

    StackExchange.using("externalEditor", function() {
    // Have to fire editor after snippets, if snippets enabled
    if (StackExchange.settings.snippets.snippetsEnabled) {
    StackExchange.using("snippets", function() {
    createEditor();
    });
    }
    else {
    createEditor();
    }
    });

    function createEditor() {
    StackExchange.prepareEditor({
    heartbeatType: 'answer',
    autoActivateHeartbeat: false,
    convertImagesToLinks: true,
    noModals: true,
    showLowRepImageUploadWarning: true,
    reputationToPostImages: 10,
    bindNavPrevention: true,
    postfix: "",
    imageUploader: {
    brandingHtml: "Powered by u003ca class="icon-imgur-white" href="https://imgur.com/"u003eu003c/au003e",
    contentPolicyHtml: "User contributions licensed under u003ca href="https://creativecommons.org/licenses/by-sa/3.0/"u003ecc by-sa 3.0 with attribution requiredu003c/au003e u003ca href="https://stackoverflow.com/legal/content-policy"u003e(content policy)u003c/au003e",
    allowUrls: true
    },
    noCode: true, onDemand: true,
    discardSelector: ".discard-answer"
    ,immediatelyShowMarkdownHelp:true
    });


    }
    });














    draft saved

    draft discarded


















    StackExchange.ready(
    function () {
    StackExchange.openid.initPostLogin('.new-post-login', 'https%3a%2f%2fmath.stackexchange.com%2fquestions%2f3076373%2fshow-that-the-lp-norms-with-1-leq-p-q-leq-infty-are-equivalent%23new-answer', 'question_page');
    }
    );

    Post as a guest















    Required, but never shown

























    1 Answer
    1






    active

    oldest

    votes








    1 Answer
    1






    active

    oldest

    votes









    active

    oldest

    votes






    active

    oldest

    votes









    1












    $begingroup$

    $|x|_{infty} leq 1$ implies $|x|_{p} leq n^{1/p}$ for any $p in [1,infty)$. Use this and a scaling argument to show that $|x|_{p} leq n^{1/p} |x|_{infty}$. [You have to consider $y=frac x {|x|_{infty}}$ when $x neq 0$]. On the other hand, $ |x|_{infty} leq |x|_{p} $ because $|x_i| leq |x|_{p} $ for each $i$. Hence all of the $p$-norms with $p<infty$ are equivalent to $infty$ norm. This implies that any two of the $p$-norms are equivalent. [ $|x|' leq c|x|''$ and $|x|'' leq d|x|'''$ implies $|x|' leq cd|x|'''$ etc].






    share|cite|improve this answer











    $endgroup$


















      1












      $begingroup$

      $|x|_{infty} leq 1$ implies $|x|_{p} leq n^{1/p}$ for any $p in [1,infty)$. Use this and a scaling argument to show that $|x|_{p} leq n^{1/p} |x|_{infty}$. [You have to consider $y=frac x {|x|_{infty}}$ when $x neq 0$]. On the other hand, $ |x|_{infty} leq |x|_{p} $ because $|x_i| leq |x|_{p} $ for each $i$. Hence all of the $p$-norms with $p<infty$ are equivalent to $infty$ norm. This implies that any two of the $p$-norms are equivalent. [ $|x|' leq c|x|''$ and $|x|'' leq d|x|'''$ implies $|x|' leq cd|x|'''$ etc].






      share|cite|improve this answer











      $endgroup$
















        1












        1








        1





        $begingroup$

        $|x|_{infty} leq 1$ implies $|x|_{p} leq n^{1/p}$ for any $p in [1,infty)$. Use this and a scaling argument to show that $|x|_{p} leq n^{1/p} |x|_{infty}$. [You have to consider $y=frac x {|x|_{infty}}$ when $x neq 0$]. On the other hand, $ |x|_{infty} leq |x|_{p} $ because $|x_i| leq |x|_{p} $ for each $i$. Hence all of the $p$-norms with $p<infty$ are equivalent to $infty$ norm. This implies that any two of the $p$-norms are equivalent. [ $|x|' leq c|x|''$ and $|x|'' leq d|x|'''$ implies $|x|' leq cd|x|'''$ etc].






        share|cite|improve this answer











        $endgroup$



        $|x|_{infty} leq 1$ implies $|x|_{p} leq n^{1/p}$ for any $p in [1,infty)$. Use this and a scaling argument to show that $|x|_{p} leq n^{1/p} |x|_{infty}$. [You have to consider $y=frac x {|x|_{infty}}$ when $x neq 0$]. On the other hand, $ |x|_{infty} leq |x|_{p} $ because $|x_i| leq |x|_{p} $ for each $i$. Hence all of the $p$-norms with $p<infty$ are equivalent to $infty$ norm. This implies that any two of the $p$-norms are equivalent. [ $|x|' leq c|x|''$ and $|x|'' leq d|x|'''$ implies $|x|' leq cd|x|'''$ etc].







        share|cite|improve this answer














        share|cite|improve this answer



        share|cite|improve this answer








        edited Jan 16 at 23:37

























        answered Jan 16 at 23:24









        Kavi Rama MurthyKavi Rama Murthy

        59k42161




        59k42161






























            draft saved

            draft discarded




















































            Thanks for contributing an answer to Mathematics Stack Exchange!


            • Please be sure to answer the question. Provide details and share your research!

            But avoid



            • Asking for help, clarification, or responding to other answers.

            • Making statements based on opinion; back them up with references or personal experience.


            Use MathJax to format equations. MathJax reference.


            To learn more, see our tips on writing great answers.




            draft saved


            draft discarded














            StackExchange.ready(
            function () {
            StackExchange.openid.initPostLogin('.new-post-login', 'https%3a%2f%2fmath.stackexchange.com%2fquestions%2f3076373%2fshow-that-the-lp-norms-with-1-leq-p-q-leq-infty-are-equivalent%23new-answer', 'question_page');
            }
            );

            Post as a guest















            Required, but never shown





















































            Required, but never shown














            Required, but never shown












            Required, but never shown







            Required, but never shown

































            Required, but never shown














            Required, but never shown












            Required, but never shown







            Required, but never shown







            Popular posts from this blog

            Mario Kart Wii

            What does “Dominus providebit” mean?

            Antonio Litta Visconti Arese